How to integrate e^x cos(x) using parts?

Click For Summary
SUMMARY

The integration of the function e^x cos(x) can be achieved using integration by parts, specifically by applying the technique twice. This results in a circular equation where the integral I can be expressed as I = something - I, leading to the conclusion I = something/2. Additionally, the problem can be approached using de Moivre's theorem, which states that cos(x) + j*sin(x) = e^(j*x).

PREREQUISITES
  • Understanding of integration techniques, specifically integration by parts.
  • Familiarity with complex numbers and de Moivre's theorem.
  • Knowledge of exponential functions and their properties.
  • Basic calculus concepts, including definite and indefinite integrals.
NEXT STEPS
  • Practice integration by parts with various functions to solidify understanding.
  • Study de Moivre's theorem and its applications in complex analysis.
  • Explore the properties of exponential functions in calculus.
  • Learn about the relationship between trigonometric functions and complex exponentials.
USEFUL FOR

Students and educators in calculus, mathematicians exploring integration techniques, and anyone interested in the application of complex numbers in integration.

Firepanda
Messages
425
Reaction score
0
Is it possible to intergrate

e^x (cosx)

i wondered because i tried to intergrate it by parts, but ended up going round in circles.

I wondered because i had this question and I am stuck on how to do it :)

http://img505.imageshack.us/img505/320/frfbc8.png
 
Last edited by a moderator:
Physics news on Phys.org
Sure it's possible. Just integrate by parts twice. You will get back to where you started - but if you call the integral I, then you will get I=somthing-I. So I=something/2. Not circular. On the other hand the question you posted is done more directly with deMoivre. cos(x)+j*sin(x)=e^(j*x).
 
Question: A clock's minute hand has length 4 and its hour hand has length 3. What is the distance between the tips at the moment when it is increasing most rapidly?(Putnam Exam Question) Answer: Making assumption that both the hands moves at constant angular velocities, the answer is ## \sqrt{7} .## But don't you think this assumption is somewhat doubtful and wrong?

Similar threads

Replies
7
Views
2K
  • · Replies 4 ·
Replies
4
Views
2K
  • · Replies 3 ·
Replies
3
Views
2K
  • · Replies 11 ·
Replies
11
Views
3K
Replies
3
Views
2K
  • · Replies 1 ·
Replies
1
Views
2K
  • · Replies 14 ·
Replies
14
Views
2K
Replies
9
Views
2K
  • · Replies 3 ·
Replies
3
Views
1K
  • · Replies 19 ·
Replies
19
Views
2K